Vous êtes sur la page 1sur 23

Page 1 of 23

SCMD 3000 GEMP 1 CARDIOVASCULAR MCQ Exam 24 MAY 2010

Venue: LT- _____ Seat No: ___________

Student No: ______________________

UNIVERSITY OF THE WITWATERSRAND, JOHANNESBURG


Faculty of Health Sciences

MB BCh III / GEMP I INTEGRATED BASIC MEDICAL AND HUMAN SCIENCES (SCMD 3000)

Block 3: Cardiovascular MCQ Exam

Venue: _____________________________________ Seat Number: _____________________________ MCQ TOTAL

Student Number: _____________________________

Time allowed: 92 minutes

Date: 24th May 2010

Page 2 of 23
SCMD 3000 GEMP 1 CARDIOVASCULAR MCQ Exam 24 MAY 2010

GENERAL INSTRUCTIONS Please read the following carefully: 1) There are 46 multiple choice questions (MCQs) in this paper. 2) Answer all MCQ questions on the MCQ cards provided 3) Write your group number, seat number and student number on each page of this question paper and on the MCQ Cards 4) Please hand in all cellphones, calculators and removable drives prior to the start of the examination. 5) You may not remove any question papers from the examination room. A-TYPE QUESTIONS Select the single best answer to each question. (Note: several options may be correct but only one is the best answer.) Answers must be entered on the question paper and on the MCQ Cards If you give more than one answer for a question, you will score zero for that question. There will be no penalty for incorrect answers. If you do not know an answer you may leave it blank, in which case you will neither earn a mark nor be penalised. X-TYPE QUESTIONS There is at least one correct statement and at least one incorrect statement. Identify BOTH the correct and incorrect statements. Answers must be entered on the question paper and on the MCQ Cards If you are unsure of a statement, leave it blank on the MCQ card Negative marking will be applied to wrong answers but negative marks will not be carried forward. This means the minimum mark for a question is zero. R-TYPE QUESTIONS Here you are offered a number of answers. This is followed by a set of questions. Select the single best answer to each question. Note that each answer may be used more than once or not at all unless you are specifically instructed otherwise. Fill in your answers in the spaces provided on the question paper and online. If you do not know an answer you may leave it blank, in which case you will neither earn a mark nor be penalised. Note: Each A-type, X-type and R-type question has the same mark value unless specified differently IMPORTANT: THERE ARE SOME A-TYPE QUESTIONS BETWEEN THE X-TYPES

Page 3 of 23
SCMD 3000 GEMP 1 CARDIOVASCULAR MCQ Exam 24 MAY 2010

Venue: LT- _____ Seat No: ___________


A-types (Answer on red MCQ Card CSX-289) A1.

Student No: ______________________

A 42-year-old woman has noted increasing dyspnoea for the past 6 years. On examination rales are auscultated in both lungs. She is afebrile. A chest radiograph shows an enlarged cardiac silhouette and bilateral pulmonary oedema. Past history reveals that, as a child she suffered recurrent bouts of pharyngitis with group A beta haemolytic streptococcal infections. Which valve or pair of valves is the most likely valve abnormality in this woman? a. Aortic and pulmonary b. Aortic and tricuspid c. Mitral alone d. Mitral and aortic e. Mitral and pulmonary

Option C & D marked correct Feedback C - CORRECT. She has chronic rheumatic valvulitis with scarring associated with rheumatic heart disease. If the tricuspid valve is involved, then the mitral and aortic are probably involved as well. The most common single valve involved is the mitral. A2. Mrs Banda is a 26 year old accountant who has a history of 4 first trimester miscarriages. She is planning another pregnancy. On clinical examination of Mrs Banda and her partner no abnormalities were detected. From the following, which genetic test would be most appropriate for Mrs Banda? a. Fluorescent in situ hybridization (FISH) analysis b. Chromosome analysis c. DNA mutation analysis d. PCR aneuploidy analysis e. MLPA microdeletion screen

Correct answer: B to look for balanced translocation which may have resulted in the multiple miscarriages. FISH, DNA analysis incorrect because need to request specific test (underlying diagnosis) Aneuploidy analysis unlikely to reveal any abnormality with normal clinical examination MLPA microdeletion not appropriate test
A3. When breaking bad news, which statement is the most important to consider: a. to only speak to patients once you have a definitive diagnosis b. have experts on hand to answer the patients questions immediately c. to give information in simple terminology d. to discuss all the relevant information in detail with the patient so that the patient is fully informed e. bad news should not be given to patients who are anxious or unwell

Option C & D marked correct Feedback C - CORRECT.

Page 4 of 23
SCMD 3000 GEMP 1 CARDIOVASCULAR MCQ Exam 24 MAY 2010 A4. A proposed research study has the following study question: What is the experience at Chris Hani Baragwanath Hospital of using thrombolysis with streptokinase during cardiopulmonary resuscitation?Which of the study designs below would be the most appropriate? a. Case-control study b. Cohort study c. Randomised controlled trial d. Cross-sectional study e. Case series

X-types (Answer on orange MCQ Card CSX-292) On receiving the news of the tragic passing of her daughter Mrs. Kapinski became dizzy and fell to the floor. The following table lists Mrs Kapinskis and a healthy persons data when asked to perform a Valsalva manoeuver. Mrs Kapinskis problem was subsequently attributed to an emotional response exacerbated by a -adrenergic receptor blocker that she was receiving at the time. Patient No Valsalva Blood pressure (mm Hg) Heart rate (beats/minute) X1. 136/82 76 Valsalva 92/42 76 Healthy person No Valsalva 134/84 76 Valsalva 122/86 92

The effect of the Valsalva manoeuver on systolic blood pressure in the healthy person is attributed to a. an increased intrathoracic pressure. b. a decreased myocardial -adrenoreceptor activation. c. increased parasympathetic nervous system activation. d. a decreased ventricular filling. e. a tachycardia. True True True True True False False False False False

Reasons a) Correct. The valsalva maneuver involves expiration against a closed glottis and hence increases intrathoracic pressure. The consequence is a decreased venous return, which depends on low intrathoracic pressures and hence a reduced cardiac filling and hence stroke volume and cardiac output. As cardiac output is reduced so is systolic blood pressure. b) Incorrect. Systolic blood has decreased in response to a reduced venous return. The compensatory increase in sympathetic nervous system activity, as indicated by the increased heart rate, will produce -receptor activation and hence an increase and not decrease in systolic blood pressure. c) Incorrect. The normal person has a compensatory increase in sympathetic nervous system rather than parasympathetic activity, as evidenced by the increased heart rate. d) Correct. As indicated in the answer to a), the valsalva maneuver increases intrathoracic pressure. The consequence will therefore be a decreased venous return. e) Incorrect. A tachycardia will increase cardiac output and an increased cardiac output will increase systolic blood pressure (BP=CO x TPR). The increase in heart rate is not sufficiently high to reduce ventricular filling.

Page 5 of 23
SCMD 3000 GEMP 1 CARDIOVASCULAR MCQ Exam 24 MAY 2010

Venue: LT- _____ Seat No: ___________


X2.

Student No: ______________________

In Mrs Kapinski, the markedly greater decrease in blood pressure as compared to the normal person when performing the Valsalva manoeuver is attributed to a. excessive -adrenoceptor activation. b. decreased baroreceptor sensitivity. c. a decreased ability of sympathetic activation to increase heart rate. d. a decreased ability of sympathetic activation to increase cardiac contractility. e. a greater decrease in cardiac filling produced by the manoeuver. True True True True True False False False False False

Reasons a) Incorrect. Although -adrenoreceptor activation will be greater than -adrenoreceptor activation in the presence of a -blocker, -adrenoreceptor activation will increase and not decrease blood pressure. b) Incorrect. There is no reason to believe that the use of -blockers will decrease baroreceptor sensitivity. c) Correct. Her heart rate shows no change in response to the valsalva maneuver and this is very likely to be because of -adrenoreceptor blockade at the SA node. Thus an increase in heart rate will not contribute to maintaining an increased cardiac output and blood pressure produced by sympathetic activation during the valsalva maneuver. d) Correct. As previously indicated, her heart rate shows no change in response to the valsalva maneuver and this is very likely to be because of -adrenoreceptor blockade at the SA node. Thus, it is likely that the same degree of -adrenoreceptor blockade will have occurred in the myocardium. Thus, an increase in myocardial contractility will not contribute to maintaining an increased cardiac output and blood pressure produced by sympathetic activation during the valsalva maneuver. e) Incorrect. For this to occur she would have had to have had a decreased blood volume or increased vascular capacitance. There is no evidence to suggest that her blood volume would be decreased or that her venous tone is markedly reduced.

Page 6 of 23
SCMD 3000 GEMP 1 CARDIOVASCULAR MCQ Exam 24 MAY 2010 The following information relates to questions X3 to X6 Two patients, whom had previously been diagnosed as having either systemic (Mr Naidoo) or pulmonary (Mr Mahlaba) hypertension, had electrocardiograph (ECG) recordings that showed electrical axis shifts. Mr Naidoo had an electrical axis in the vertical plane leads of -60o and Mr Mahlaba an electrical axis in the vertical plane leads of +120o. Special investigations revealed a markedly enlarged left ventricle in Mr Naidoo and markedly enlarged right ventricle in Mr Mahlaba. X3. Mr Naidoos ECG recording is likely to show a. an equiphasic QRS complex in lead aVR. b. a negative QRS complex in lead aVL. c. an increased R wave of the QRS complex in lead V5. d. a positive QRS complex in lead V1. e. a negative QRS complex in lead III. True True True True True False False False False False

Reasons a) Correct. With an electrical axis of -60 degrees, the lead that lies at 90 degrees to this axis, and hence the lead that will show an equiphasic recording, is lead aVR. b) Incorrect. Lead aVL lies at -30 degrees and this is close to the electrical axis. This lead will therefore show a positive QRS complex. c) Correct. With marked left ventricular hypertrophy, the electrical axis in horizontal plane leads may not shift but there will be a greater degree of conduction through the left ventricular wall. This the R wave increases in amplitude. d) Incorrect. As the electrical axis in the horizontal plane does not change in left ventricular hypertrophy, lead V1 which faces the right ventricle, will continue to produce a negative QRS complex. e) Correct. The electrical axis faces away from lead III (120 degrees). Thus, this lead will show a large negative QRS complex. X4. Mr Mahlabas ECG recording is likely to show a. an equiphasic QRS complex in lead aVR. b. a negative QRS complex in lead aVL. c. an positive QRS complex in lead V5. d. a positive QRS complex in lead V1. e. a negative QRS complex in lead III. True True True True True False False False False False

Reasons a) Correct. With an electrical axis of +120 degrees, the lead that lies at 90 degrees to this axis, and hence the lead that will show an equiphasic recording, is lead aVR. b) Correct. Lead aVL lies at more than +90 degrees to the electrical axis and hence will show a negative QRS complex. c) Incorrect. With marked right ventricular hypertrophy, the electrical axis in horizontal plane leads will be toward the right ventricle and away from the left ventricle. In this instance, lead V5 which faces the left ventricle will be a negative QRS complex. d) Correct. As indicated above with marked right ventricular hypertrophy, the electrical axis in horizontal plane leads will be toward the right ventricle and away from the left ventricle. In this instance, lead V1 which faces the right ventricle will be a negative QRS complex. e) Incorrect. The electrical axis faces lead III (120 degrees). Thus, this lead will show the largest positive QRS complex.

Page 7 of 23
SCMD 3000 GEMP 1 CARDIOVASCULAR MCQ Exam 24 MAY 2010

Venue: LT- _____ Seat No: ___________


X5.

Student No: ______________________

Mr. Naidoo, who had severe uncontrolled systemic hypertension had a chest X-ray that showed upper lobe blood diversion, peribronchial cuffing, fluid in the fissures, a cardiothoracic ratio of 75%, an unfolded aorta, and an enlarged left atrium. Echocardiography revealed a left ventricle with function and geometry that suggested the presence of diastolic heart failure. The chest X-Ray features described above for Mr. Naidoo are consistent with a. right heart failure. b. a dissecting aortic aneurysm. c. increased pressures in the left atrium. d. pulmonary congestion. e. left heart failure. True True True True True False False False False False

Reasons a) Incorrect. Upper lobe blood diversion, peribronchial cuffing, and fluid in the fissures are signs of left and not right heart failure. b) Incorrect. An unfolded aorta is a sign of high loads or pressures in the aorta and not of aortic dissection. c) Correct. As previously indicated upper lobe blood diversion, peribronchial cuffing, and fluid in the fissures are signs of left and not right heart failure. This occurs as a consequence of increased left atrial pressures. Moreover an enlarged left atrium is also consistent with increased pressures in the left atrium. d) Correct. Left heart failure produces pulmonary oedema which tends to occur in the lung bases following gravity effects. The consequence is that blood in the lung bases is hypoxic and vasoconstriction occurs in this pulmonary region. Blood is therefore shunted to the upper lobes of the lung. Moreover, peribronchial cuffing and fluid in the fissures are early signs of pulmonary oedema. e) Correct. See answers to question c. X6. Echocardiographic evidence of diastolic heart failure in Mr. Naidoo may include a a. dilated left ventricle. b. preserved left ventricular ejection fraction despite evidence of heart failure. c. thin-walled left ventricle. d. concentric cardiac geometry (increased wall thickness-toradius ratio). e. high cardiac output. True True True True True False False False False False

Reasons a) Incorrect. Diastolic heart failure is associated with a stiff ventricular wall and hence will have a normal or decreased left ventricular chamber diameter. Only systolic heart failure will produce a dilated ventricle through adverse remodeling. b) Correct. In the context of a patient with left heart failure, as is the case in Mr Naidoo, a normal left ventricular pump function is evidence for the presence of diastolic heart failure. c) Incorrect. As indicated in the answer to a, diastolic heart failure is associated with a stiff ventricular wall. This is unlikely to occur if the wall thins. d) Correct. Diastolic heart failure is associated with a stiff ventricular wall and hence will have a normal or decreased left ventricular chamber diameter, whilst wall thickness will be increased. This is the definition of a concentric geometry.

Page 8 of 23
SCMD 3000 GEMP 1 CARDIOVASCULAR MCQ Exam 24 MAY 2010 e) Incorrect. Although pump function may be normal or even increased in diastolic heart failure, cardiac output is not enhanced. This would only occur in high output heart failure. The following information relates to questions X7 and X8 Mr Thompson experienced acute chest pain and was immediately admitted to hospital. On examination his blood pressure was 92/56 mm Hg, his heart rate was 136 beats per minute and he was pale, his skin was cold to the touch, he was sweating and nauseous and he had a raised jugular venous pressure, but no evidence of pulmonary congestion. His electrocardiograph recording showed ST segment elevation in leads V2 to V6 in the horizontal plane and leads aVL and I in the vertical plane of the heart, as well as ST segment depression in leads II, aVF, and III. X7. Mr Thompsons ECG recording indicates the presence of a. left ventricular hypertrophy. b. transmural myocardial ischaemia in the anterior wall of the left ventricle. c. subendocardial ischaemia in the lateral wall of the heart. d. transmural myocardial ischaemia in the inferior wall of the heart. e. an increased permeability of cardiomyocyte cell membranes to K+ ions. True True True True True False False False False False

Reasons a) Incorrect. Although left ventricular hypertrophy may produce ST segment depression, this occurs in lateral wall leads (lead I, V4-V6). b) Correct. Transmural ischaemia produces ST segment elevation in leads facing the ischaemic zone. As V2 and V3 show ST segment elevation and these leads face the anterior wall of the left ventricle, this indicates that transmural ischaemia exists in the anterior wall of the left ventricle. c) Incorrect. The lateral wall leads, V4, V5, V6, and I all show ST segment elevation. This indicates transmural rather than subendocardial ischaemia. d) Incorrect. The inferior wall leads, II, III and aVF all show ST segment depression. This indicates subendocardial rather than transmural ischaemia. e) Correct. The charge difference between ischaemic and normal tissue that cause ST segment changes is generated by leakiness of the cardiomyocyte to K+ ions. X8. Mr Thompson has clinical signs consistent with a. marked vasodilation. b. cardiogenic shock. c. hypovolaemic shock. d. high output heart failure. e. an increased sympathetic nervous system activity. True True True True True False False False False False

Reasons a) Incorrect. Marked vasodilation leading to shock will result in a plethoric rather than a pale appearance and filling pressures in the heart and hence jugular venous pressure will be reduced and not increased. b) Correct. Mr Thompson has a raised jugular venous pressure and occlusion of the left anterior descending coronary artery. The myocardial ischaemia would cause marked increases in left atrial pressures, increased pulmonary artery pressures, an increased

Page 9 of 23
SCMD 3000 GEMP 1 CARDIOVASCULAR MCQ Exam 24 MAY 2010

Venue: LT- _____ Seat No: ___________

Student No: ______________________

afterload to the right ventricle and hence a raised jugular venous pressure. The reduced blood pressures indicate the presence of shock. c) Incorrect. Mr Thompson has a raised jugular venous pressure and hence evidence of increased cardiac filling pressures. Hypovolaemic shock will decrease cardiac filling pressures. d) Incorrect. High output heart failure would result in a high systolic blood pressure, a widened pulse pressure and would not show the sympathetic response noted in the patient. e) Correct. Mr Thompson has a number of symptoms and signs that indicate sympathetic over-activation including signs of vasoconstriction (pallor and cold skin), sweating, a tachycardia, and nausea. The following information relates to questions A5 and X9 to X11 Mrs Sibeko, a 65 year old lady with a 20 year history of hypertension consulted her doctor. Mrs Sibeko complained that she frequently felt dizzy especially when standing up quickly and that she felt short of breath when walking up an incline. Her blood pressure (mean of three measurements in the seated position off medication) was 160/92 mm Hg and an additional heart sound (S3 or S4) was heard. She was noted to have pulmonary oedema, a raised jugular venous pressure and left ventricular hypertrophy, but no cardiac murmurs were detected. Her ejection fraction was normal but her left ventricular end diastolic diameter was decreased. A-type question (answer on red MCQ Card CSX-289) A5. Mrs Sibekos dizziness and shortness of breath are likely to be due to decreased a. blood volume b. left ventricular filling c. cardiac contractility d. peripheral vasoconstriction e. afterload

Answers: a) incorrect, no evidence of blood loss b) correct, left ventricular hypertrophy could cause diastolic dysfunction and hence decreased filling, a reduced stroke volume and hence symptoms of dizziness and shortness of breath c) incorrect, decreased contractility would result in decreased ejection fraction d) incorrect, decreased peripheral vasoconstriction unlikely as her DBP is increased (vasodilation increases venous return) e) incorrect, a decreased afterload would increase stroke volume (she is unlikely to have a decreased afterload as she has hypertension)

Page 10 of 23
SCMD 3000 GEMP 1 CARDIOVASCULAR MCQ Exam 24 MAY 2010 X-type questions (answer on orange MCQ Card CSX-292) X9. a. b. c. d. e. Mrs Sibekos raised jugular venous pressure is likely to result from an increased right atrial pressure from an increased right ventricular afterload from a decreased pulmonary vascular resistance in hepatomegaly in a decreased systemic capillary hydrostatic pressure True True True True True False False False False False

Answers: a) correct, if right atrial pressure is increased then jugular venous pressure would be increased as there is no valve between the right atrium and the jugular vein b) correct, an increased right ventricular afterload (which is likely as she has pulmonary oedema) would result in decreased stroke volume and hence increased right atrial pressure c) incorrect, with pulmonary oedema hypoxic hypoxia would result which would cause pulmonary vasoconstriction and hence an increased pulmonary vascular resistance d) correct, increased right atrial pressure results in decreased venous return and hence venous congestion in the liver, resulting in an enlarged liver e) incorrect, a raised jugular venous pressure is likely to cause an increased systemic capillary hydrostatic pressure X10. Mrs. Sibekos heart failure is likely to be due to a True True True True True False False False False False

a. thiamine deficiency. b. prior myocardial infarction. c. stiff (non-compliant) left ventricle. d. high afterload to the left ventricle. e. reduced myocardial contractility.

Answers: a) incorrect, a thiamine deficiency would result in an increased venous return and hence an increased left ventricular end diastolic diameter. b) incorrect, she has a normal ejection fraction c) correct, she has a decreased left ventricular end diastolic diameter d) correct, she has a history of hypertension and left ventricular hypertrophy therefore she is likely to have an increased left ventricular afterload e) incorrect, she has a normal ejection fraction X11. The additional heart sound (S3 or S4) in Mrs. Sibeko is likely to be due to True True True True True False False False False False

a. pericardial effusion. b. increased left ventricular filling pressures. c. high velocity of blood flow in the left ventricle during diastole. d. reduced capacity of the left ventricle to fill during early diastole. e. mitral stenosis.

Answers: a) incorrect, a pericardial effusion produces a friction rub which is generally heard throughout the cardiac cycle.

Page 11 of 23
SCMD 3000 GEMP 1 CARDIOVASCULAR MCQ Exam 24 MAY 2010

Venue: LT- _____ Seat No: ___________

Student No: ______________________

b) correct, she has heart failure and hence increased filling pressures, which would impede early left ventricular filling (passive filling) and hence increase the reliance on atrial contraction for filling which would increase the velocity and hence turbulence of blood flow across the mitral valve. c) correct, as the heart has increased reliance on atrial contraction for filling (see b above), the velocity and hence turbulence of blood flow across the mitral valve is increased giving rise to the additional heart sound d) correct, she has left ventricular hypertrophy and a decreased left ventricular end diastolic diameter. Also she has heart failure and hence increased filling pressures, which would impede early left ventricular filling (passive filling) and hence increase the reliance on atrial contraction for filling which would increase the velocity and hence turbulence of blood flow across the mitral valve. e) incorrect, a mitral stenosis would result in an diastolic murmur (she has no cardiac murmurs) X12. Mr. Mbelwa is a 43 year old male patient with a 10 year history of high blood pressure. He later diagnosed with diabetes mellitus and he had also developed vascular remodeling. You are concerned about the development of atherosclerosis in Mr. Mbelwa because hypertension is associated with: a. decreased oxidative stress b. platelet aggregation c. decreased collagen production d. smooth muscle cell proliferation e. increased plasma HDL cholesterol concentration True True True True True False False False False False

Answers: a) incorrect, diabetes mellitus is a contraindication for the use of beta blockers b) correct, c) incorrect, as the patient does not have blood pressures >180/110 mm Hg d) correct, he has diabetes mellitus and hence increased cardiovascular risk, hence he has a compelling indication for the use of an ACE inhibitor. e) incorrect, as Mr Mbelwa has increased cardiovascular risk and hence complicated hypertension which warrants the use of pharmacological agents X13. In myocardial ischaemia, myocyte True True True True True False False False False False

a. metabolite washout is increased. b. oxygen delivery is reduced. c. Na /K -ATPase pump activity is increased. d. adenosine production is increased. e. lactate production is increased.
+ +

Reasons: a) Incorrect. Decreased myocardial blood flow reduces washout of metabolic waste. b) Correct. Reduced myocardial blood flow reduces oxygen delivery to myocytes. c) Incorrect. A lack of ATP decreases activity of the Na+/K+-ATPase pump and this results in ion imbalances in the myocyte. d) Correct. In ischaemia adenosine is produced by the sequential breakdown of ATP to ADP to AMP and eventually to adenosine. e) Correct. A lack of oxygen delivery increases lactate production by anaerobic glycolysis.

Page 12 of 23
SCMD 3000 GEMP 1 CARDIOVASCULAR MCQ Exam 24 MAY 2010 A-type question (answer on red MCQ Card CSX-289) A6. In the cardiac cycle a. the second heart sound occurs at the beginning of ventricular systole. b. peak ventricular pressure equals systolic blood pressure.

c. the P-wave of an ECG coincides with the beginning of isovolumetric ventricular contraction. d. atrial contraction increases ventricular end-diastolic volume by more than 50%. e. the mitral valve is open during isovolumetric ventricular relaxation.

Answers: a) incorrect - the second heart sound occurs at the end of ventricular systole. b) correct - systolic blood pressure is determined by peak ventricular pressure. c) incorrect - the P-wave of an ECG coincides with atrial contraction. d) incorrect - atrial contraction increases ventricular end-diastolic volume by 20-30%. e) incorrect no valves are open during isovolumetric ventricular relaxation. X-type questions (answer on orange MCQ Card CSX-292) X14. In aortic incompetence True True True True True False False False False False

a. a systolic murmur occurs. b. pulse pressure increases. c. blood flows from the aorta to the left ventricle. d. left ventricular filling volume decreases. e. exercise intolerance occurs. Answers: a) incorrect - a diastolic murmur occurs.

b) correct - pulse pressure increases because systolic pressure is increased and diastolic pressure is much lower than normal c) correct - blood flows from the aorta to the left ventricle because the aortic valve stays open in diastole d) incorrect - left ventricular filling volume increases because of blood flowing back from the aorta to the left ventricle e) correct exercise intolerance may occur due to insufficient blood supply to (or perfusion of) the tissues because blood moves back to the left ventricle.

Page 13 of 23
SCMD 3000 GEMP 1 CARDIOVASCULAR MCQ Exam 24 MAY 2010

Venue: LT- _____ Seat No: ___________


X15.

Student No: ______________________

Cardiac output is increased by an increase in True True True True True False False False False False

a. end diastolic volume. b. parasympathetic activity. c. right ventricular end-diastolic pressure. d. ventricular contractility. e. circulating adrenaline. Answers:

a) correct - increased EDV increases SV which is a determinant of CO (CO=HR x SV) b) incorrect CO is increased by an increase in SNS activity (not PNS). c) incorrect increase RVEDP increases after-load and decreases CO. d) correct - increased contractility increases SV which is a determinant of CO e) correct increased adrenaline increases SNS activity and therefore increases HR, which is a determinant of CO. X16. Renin True True True True True False False False False False

a. is secreted by cells of the juxtaglomerular complex. b. converts angiotensin I to angiotensin II. c. secretion is increased by sympathetic nervous system activity. d. secretion is increased in response to a decrease in blood pressure. e. secretion decreases plasma angiostensinogen levels. Answers:

a) correct - renin is secreted by juxtaglomerular cells following a decreased amount of NaCl in tubular fluid that reaches the distal tubule. b) incorrect renin converts angiotensinogen to angiotensin I. c) correct there are both direct and indirect stimulatory SNS effects on renin production. The direct action is probably via - adrenoreceptors, whereas the indirect effect is due to the SNS-mediated increased reabsorption of sodium at the proximal tubule resulting in a reduced delivery of sodium to the juxtaglomerular complex, which increases renin secretion. d) correct in response to decreased BP, the renal barorecptors cause increased SNS activity which directly increases secretion of renin. (see c) above) e) correct

Page 14 of 23
SCMD 3000 GEMP 1 CARDIOVASCULAR MCQ Exam 24 MAY 2010 X17. a. b. c. d. e. Features suggestive of Somatisation include: An acute course of symptoms Symptoms in multiple organ systems Absence of other psychiatric disorders Multiple vague symptoms Doctor hopping True True True True True False False False False False

X18.

In a patient with raised cardiac biomarkers: True True True True True False False False False False

a. Coronary reperfusion can be considered successful when the peak of the serum biochemical marker occurs early b. Measured cardiac Troponin I is a subunit of the regulatory troponin complex c. Myoglobin is an excellent Rule in marker for acute myocardial infarction d. Raised myoglobin is specific for myocardial injury

e. Cardiac Troponin T is an excellent Rule out marker for acute myocardial infarction

FEEDBACK A True due to the wash out phenomenon B True C False it is not specific for AMI and is an excellent Rule out marker D False not specific but sensitive E True X19. a. Regarding the histology of muscular arteries and veins, Muscular arteries have a prominent internal elastic lamina. True True True False False False

b. The tunica media of a muscular artery contains many elastic lamellae. c. Muscular veins have a thinner tunica adventitia than tunica media. Option D Removed e. The tunica media of a muscular vein is thicker than that of a muscular artery.

True

False

Page 15 of 23
SCMD 3000 GEMP 1 CARDIOVASCULAR MCQ Exam 24 MAY 2010

Venue: LT- _____ Seat No: ___________


X20.

Student No: ______________________

Regarding beta blockers used in cardiovascular conditions: True True True True True False False False False False

a. atenolol is used in the treatment of peripheral vascular disease b. carvedilol is an alpha-1-adrenoceptor antagonist c. propranolol is a cardioselective beta-adrenoceptor antagonist d. propranolol causes bronchoconstriction in asthmatics e. atenolol inhibits the release of renin from Juxtaglomerular cells Feedback atenolol is contraindicated in peripheral vascular disease propranolol blocks beta-1- and beta-2-adrenoreceptors X21. Regarding cardiovascular drugs:

a. c.

a. isosorbide mononitrate decreases venous capacitance b. adrenaline is an agonist at beta-1-adrenoceptors c. digoxin inhibits the activity of Na+/K+-ATPase d. nitric oxide stimulates the activity of guanylate cyclase e. sildenafil decreases cGMP levels in vascular smooth muscle Feedback isosorbide mononitrate increases venous capacitance sildenafil increases cGMP levels X22. In the treatment of cardiac conditions:

True True True True True

False False False False False

a. e.

a. glyceryl trinitrate decreases blood pressure b. amlodipine is an angiotensin receptor antagonist c. hydralazine increases venous capacitance d. nifedipine increases calcium entry into the myocardium e. valsartan is an alpha-1-adrenoceptor antagonist Feedback amlodipine is a calcium channel blocker nifedipine blocks calcium entry

True True True True True

False False False False False

b. d.

Page 16 of 23
SCMD 3000 GEMP 1 CARDIOVASCULAR MCQ Exam 24 MAY 2010 X23. Regarding cardiac drugs: True True True True True False False False False False

a. furosemide competitively inhibits the binding of aldosterone to its receptor b. amiodarone is used to prevent ventricular arrhythmias c. digoxin causes ventricular arrhythmia as an adverse effect d. lignocaine is a sodium channel blocker e. digoxin slows AV-nodal conduction Feedback its a potassium sparing diuretic X24.

a.

Which of the following molecular techniques can be used to detect and characterize subtle (submicroscopic) structural chromosome abnormalities True True True True True False False False False False

a. Comparative genomic hybridization (CGH) b. Fluorescence in situ hybridization (FISH) c. PCR aneuploidy screen d. Northern blot e. DNA sequencing

X25.

Which of the following are famous firsts in Cardiology? True True True True True False False False False False

a. William Harvey, an English physician, first describes blood circulation. b. Rene Laennec, a French physician, invents the stethoscope.

c. Willem Beethoven, a Dutch physiologist, develops the electrocardiograph d. Christiaan Barnard, a South African surgeon, performs the first whole heart transplant from one person to another e. Michael DeBakey, a South African surgeon, implants a permanent artificial heart into a patient.

Page 17 of 23
SCMD 3000 GEMP 1 CARDIOVASCULAR MCQ Exam 24 MAY 2010

Venue: LT- _____ Seat No: ___________


X26.

Student No: ______________________

Which of the following statements is/are true regarding the South African health care system? True True True True True False False False False False

a. The South African government wants to centralize health care services b. Primary health care is offered in district hospitals

c. Tertiary hospitals refer patients up to regional hospitals for more specialized care. d. The health care system includes specialized hospitals e.g. TB hospitals e. Primary health care is delivered through the district health care system.

R-type questions (answer on orange MCQ Card CSX-292) Mrs Tshabalala has been feeling unusually tired for the past few weeks. OPTIONS A. Reason for encounter B. C. D. E. F. G. H. I. J. K. L. Limit of tolerance Biopsychosocial approach Illness Limit of anxiety Folk sector Popular sector Professional sector Temporalising Sanctioning Sick role Patient centeredness

Page 18 of 23
SCMD 3000 GEMP 1 CARDIOVASCULAR MCQ Exam 24 MAY 2010

For each of the following statements regarding the scenario, choose the single best answer from the list above. Each option may only be used once. R1. She tells her husband about her tiredness. He says shes probably been working too hard and suggests she spend the weekend just relaxing at home. A B C D E F G H I J K L

R2. She spends the weekend relaxing but the tiredness continues. She goes to a pharmacy in the nearest shopping mall and asks the pharmacy nurse what she should do about her fatigue and loss of energy. Option D & H marked correct A B C D E F G H I J K L

A. B. C. D. E. F. G. H. I. J.

Right coronary artery Left coronary artery Right marginal branch Posterior interventricular artery Atrioventricular nodal branch Sinoatrial nodal branch Anterior interventricular artery Diagonal branch Left marginal branch Left circumflex artery

Regarding the coronary arteries: R3. Occlusion of this vessel results in infarction of part of the posterior and the inferior wall of the left ventricle, as well as the right ventricular free wall of the heart. A B C D E F G H I J

A. B. C. D. E. F.

Truncus arteriosus Primitive ventricle Primitive atrium Sinus venosus Aorticopulmonary septum Anterior truncal ridge

Regarding the embryological development of the heart R4. This structure contributes to the membranous part of the interventricular septum. A B C D E F

Page 19 of 23
SCMD 3000 GEMP 1 CARDIOVASCULAR MCQ Exam 24 MAY 2010

Venue: LT- _____ Seat No: ___________


A. B. C. D. E. F. G. H. I. J.

Student No: ______________________

Contraction band necrosis Coronary thrombosis Extensive transmural collagen deposition Lymphocytic interstitial infiltrates Myocarditis Myofibre disarray Necrosis with neutrophils Oedema and loss of cross striations Pericardial tamponade Perivascular and interstitial amyloid deposition

For each of the scenarios below identify the most likely histopathological finding from the list A to J above. R5. Question Removed R6. A 60-year-old man had chest pain and was hospitalized. On the first day of admission, a coronary angiogram revealed 75% stenosis of the left anterior descending artery. Four days later he suddenly becomes worse, with marked hypotension. A pericardiocentesis is performed and returns 150ml of bloody fluid. He dies, despite appropriate resuscitative measures. Which of the microscopic findings above is most likely to be present in his left ventricular myocardium at the time of his death? A B C D E F G H I J Feedback: Necrosis with neutrophils CORRECT. He has had an acute myocardial infarction complicated by rupture. This is a typical complication about 3 to 5 days following the onset. 75% arterial narrowing is the point at which coronary occlusion becomes very serious.

Page 20 of 23
SCMD 3000 GEMP 1 CARDIOVASCULAR MCQ Exam 24 MAY 2010

A. B. C. D. E. F. G. H. I. J. K. L.

Aortic coarctation Atherosclerotic aortic aneurysm Cardiomyopathy Coronary atherosclerosis Marantic endocarditis Mitral valve stenosis Mnckeberg's medial calcinosis Polyarteritis nodosa Rheumatic fever Superior mesenteric artery thrombosis Systemic lupus erythematosus Takayasu arteritis

For each of the scenarios below identify the most likely diagnosis from the list A to L above. R7. A 45-year-old woman has had worsening shortness of breath for 3 years. She now has to sleep sitting up on two pillows. She has had difficulty swallowing for the past year. She has no history of chest pain. A month ago, she had a "stroke" with resultant inability to move her left leg and difficulty moving her left arm. She is afebrile. A chest radiograph reveals a near-normal left ventricular size with a prominent left atrial border. A B C D E F G H I J K L Feedback Mitral valve stenosis CORRECT. Mitral valve stenosis leads to left atrial enlargement, but the left ventricle is usually small. THere is typically a 'fishmouth' shaped mitral valve that has stenosis as well as insufficiency, since it does not close completely. Most mitral valvular disease in adults results from rheumatic valvulitis. The episode(s) of rheumatic fever occurred years before R8. A 65-year-old man has sudden onset of severe abdominal pain. Physical examination reveals his temperature is 37C, heart rate 110/minute, respirations 25/minute, and blood pressure 145/100 mmHg. He has diminished pulses in the lower extremities. There is a pulsatile abdominal mass. A B C D E F G H I J K L Feedback Atherosclerotic aortic aneurysm CORRECT. The aorta involved with an atherosclerotic aneurysm is markedly enlarged and filled with thrombus. Risk factors for atherosclerosis include both diabetes mellitus and hypertension. Atherosclerotic aortic aneurysms are typically located in the abdominal portion below the renal arteries.

Page 21 of 23
SCMD 3000 GEMP 1 CARDIOVASCULAR MCQ Exam 24 MAY 2010

Venue: LT- _____ Seat No: ___________


A. B. C. D. E. F. G.

Student No: ______________________

Acute fibrinous pericarditis Acute myocardial infarction Dilated cardiomyopathy Myocardial rupture Non-bacterial thrombotic endocarditis Pulmonary hypertension Thrombo-embolism

For each of the following scenarios which of the complications in the list A to G above is most likely to occur? R9. A 2-year-old child has had failure to thrive for a year, becoming increasingly listless. On examination she is found to have a soft, rumbling systolic ejection murmur. An echocardiogram reveals a large membranous ventricular septal defect. Which of the complications is she most likely to experience as an adult 2 decades later if this lesion remains untreated? A B C D E F G Feedaback Pulmonary hypertension CORRECT. The left-to-right shunt eventually leads to pulmonary hypertension and reversal of the shunt (Eisenmenger complex). R10. Question Removed

A. B. C. D. E. F. G. H.

Bicuspid aortic valve Coarctation of the aorta Dextrocardia Hypertrophic sub-aortic stenosis Hypoplastic left heart syndrome Pulmonary valve stenosis Tetralogy of Fallot Ventricular septal defect

For the following scenario identify the most likely congenital abnormality in the list A to G above R11. A 20-year-old primigravida delivers a term baby girl following an uncomplicated pregnancy. No anomalies are noted at the time of birth. Five weeks later, the mother brings the baby to the clinic because she has difficulty breathing and occasionally turns pale. On physical examination a pansystolic murmur is audible. Which of the following congenital cardiac anomalies is most likely to be present in this infant? A B C D E F G H Feedback Ventricular septal defect CORRECT. The most common cardiac defect is a VSD. The baby may first become symptomatic when the pulmonary arteries dilate after the first month of life and the shunting from left-to-right increases.

Page 22 of 23
SCMD 3000 GEMP 1 CARDIOVASCULAR MCQ Exam 24 MAY 2010 A. B. C. D. E. F. Maternal serum alpha-fetoprotein Amniocentesis Cordocentesis Ultrasound Nuchal translucency scan Chorionic Villus Sampling

For the following scenario, select the most appropriate prenatal test to offer the patient: R12. A woman is referred to the Genetics Clinic to discuss fetal abnormalities seen on ultrasound examination at 27 weeks of pregnancy. The fetal medicine specialist advises testing to determine whether the fetus has a chromosome abnormality. A B C D E F

A. B. C. D. E.

Malformation Deformation Disruption Dysplasia Sequence

For each patient with a birth defect select the mechanism most likely to be the underlying cause R13. In a twin pregnancy the first twin is noted to be normal but the second twin has bilateral club feet, contractures at the wrists and fingers and an asymmetrical head A B C D E

Page 23 of 23
SCMD 3000 GEMP 1 CARDIOVASCULAR MCQ Exam 24 MAY 2010

Venue: LT- _____ Seat No: ___________


A. B. C. D. E. Balanced translocation Duplication Deletion Numerical abnormality Unbalanced translocation

Student No: ______________________

R14. Which of the above types of chromosome abnormality best describes the following karyotype?

Vous aimerez peut-être aussi